Anda di halaman 1dari 17

1.

The right triangle with the largest area will be an isosceles right triangle (where both the base and height are of equal length). Therefore, given the length of diagonal AB = 10, we can determine the largest possible area of triangle ABE by making it an isosceles right triangle. That important side ratio is where the two 1's represent the two legs (the base and

the height) and represents the diagonal. Thus if we are to construct an isosceles right triangle with a diagonal of 10, then, using the side ratios, we can determine that each leg will have a length of .

Now, we can calculate the area of this isosceles right triangle:

Since an isosceles right triangle will yield the maximum possible area, we know that 25 is the maximum possible area of a right triangle with a diagonal of length 10. Therefore we are able to answer our original question: Is the area of triangle ABE greater than 25? NO it is not greater than 25, because the maximum area is 25. Since we can answer the question using Statement (2) alone, the correct answer is B. 2.

(1): 5z = 2, z = 2/5; 5z = 10, z = 2 NS (2): 3z = 2, z = 2/3; 3z = 6, z = 2 NS

Combining: Subtract the two: 2z = even even = even so z has to be an integer if z is an integer and 5z is even, z has to be even. Ans. (C) 3. Total avg of Manager and Director = x For Managers , Salary Avg = m = (x-5000), Managers Count = M For Directors , Salary Avg = d = (x+15000), Directors Count = D We have a clue in stmt that they are talking abt averages so lets substitute in the formula Avg (x) = {M(x-5000) + D(x+15000) } / (M+D) After solving the above eqn you will get a relationship between M and D and it is M/D =1/3 and hence the answer is C.

Concept of weighted averages 5000-------- Av ------------------150000 salarys are the ratio M/D = 5000 / 15000 = 1/3 the number of mangers and directors will be in the inverse ratio M / D = 3/1 so directors will be 25% always.

4. What is the sum of the digits of the positive integer n where n < 99? (1) n is divisible by the square of y. (2) y4 is a two-digit positive odd integer. It seems that (1) and (2) combined are enough to solve this question: y could be only equal to 3 or -3 so n will be divisible by 9 and 9, 18, 27, 36, 45, 54, 63, 72, 81, 90 all have a sum of digits as 9 so we have a unique answer. WRONG!!! Imagine y = 3, 5, 7, 3, etc. So y4 = 9 or 25 or 49 and y2 = 3 or 5 or 7 or 9 so n may be divisible by 3 or 5 or 7 or 9 we are not sure what the sum of the digits of n will be suppose n = 21 or 25 we get different answers for the sum of digits as 3 or 7 Not sufficient. Answer E. 5. (1): x3 could be 11 or 27 or 97 so not a unique value

(2): x4 could be 11 or 27 or 97 so not a unique value (also, x could be positive or negative) Combining: x4 = x3 x. If x3 and x4 both are integers, x cannot be irrational. X can be only 3. Ans. (C)

6. (1) Any number of stamps could be purchased (5, 5), (10, 10), (100, 100) etc. INSUFFICIENT. (2) The total value of the $0.15 stamps must be a dollar amount that ends in 5 or 0 (in the units cents position). In order for the total value of both stamps to equal $4.40, therefore, the total value of the $0.29 stamps must also be a dollar amount that ends in 5 or 0. That would only occur if a multiple of 5 $0.29 stamps are purchased. 5 $0.29 stamps = $1.45, leaving $2.95 to make $4.40. But $2.95 is not a multiple of $0.15 -- no good. 10 $0.29 stamps = $2.90, leaving $1.50 to make $4.40. So 10 $0.15 would be purchased. 15 $0.29 stamps = $4.35, leaving $0.05 to make $4.40. Clearly not a multiple of $0.15 -- no good. The only possibility is that 10 of each stamp are purchased. SUFFICIENT. The correct answer is B.

7. (1) INSUFFICIENT: This expression provides only a range of possible values for x x could have infinite values. For example, x can be -1, 0, and 1. (2) SUFFICIENT: Absolute value problems often -- but not always -- have multiple solutions because the expression within the absolute value bars can be either positive or negative even though the absolute value of the expression is always positive. For example, if we consider the equation |2 + x| = 3, we have to consider the possibility that 2 + x is already positive and the possibility that 2 + x is negative. If 2 + x is positive, then the equation is the same as 2 + x = 3 and x = 1. But if 2 + x is negative, then it must equal -3 (since |-3| = 3) and so 2 + x = -3 and x = -5. So in the present case, in order to determine the possible solutions for x, it is necessary to solve for x under both possible conditions. For the case where x > 0: x = 3x 2 -2x = -2 x=1

For the case when x < 0: x = -1(3x 2) We multiply by -1 to make x equal a negative quantity. x = 2 3x 4x = 2 x = 1/2 Note however, that the second solution x = 1/2 contradicts the stipulation that x < 0, hence there is no solution for x where x < 0. Therefore, x = 1 is the only valid solution for (2). The correct answer is B. 8. This is an AP common difference either positive or negative. There are 15 terms, so the 8th term will be the median. 7 terms will be less than the median and 7 terms will be more than the median. If median is 10, then we know that 7 terms are more than 10 and 7 terms are less than 10. Ans. B 9. We can first simplify the exponential expression in the question: ba+1 bab b(ba) - b(ab) b(ba - ab) So we can rewrite this question then as is b(ba - ab) odd? Notice that if either b or ba - ab is even, the answer to this question will be no. (1) SUFFICIENT: If we simplify this expression we get 5a - 8, which we are told is odd. For the difference of two numbers to be odd, one must be odd and one must be even. Therefore 5a must be odd, which means that a itself must be odd. To determine whether or not this is enough to dictate the even/oddness of the expression b(ba - ab), we must consider two scenarios, one with an odd b and one with an even b: a b b(ba - ab) odd/even 3 1 1(13 - 31) = -2 3 2 2(23 - 32) = -2 even even

It turns out that for both scenarios, the expression b(ba - ab) is even. (2) SUFFICIENT: It is probably easiest to test numbers in this expression to determine whether it implies that b is odd or even. b b3 + 3b2 + 5b + 7 odd/even 2 23 + 3(22) + 5(2)+ 7 = 37 1 13 + 3(12) + 5(1) + 7 = 16 odd even

We can see from the two values that we plugged that only even values for b will produce odd values for the expression b3 + 3b2 + 5b + 7, therefore b must be even. Knowing that b

is even tells us that the product in the question, b(ba - ab), is even so we have a definitive answer to the question. The correct answer is D, EACH statement ALONE is sufficient to answer the question.

10. There is no obvious way to rephrase this question. Note that x! is divisible by all integers up to and including x; likewise, x! + x is definitely divisible by x. However, it's impossible to know anything about x! + x + 1. Therefore, the best approach will be to test numbers. Note that since the question is Yes/No, all you need to do to prove insufficiency is to find one Yes and one No. (1) INSUFFICIENT: Statement (1) says that x < 10, so first we'll consider x = 2. 2! + (2 + 1) = 5, which is prime. Now consider x = 3. 3! + (3 + 1) = 6 + (3 + 1) = 10, which is not prime. Since we found one value that says it's prime, and one that says it's not prime, statement (1) is NOT sufficient. (2) INSUFFICIENT: Statement (2) says that x is even, so let's again consider x = 2: 2! + (2 + 1) = 5, which is prime. Now consider x = 8: 8! + (8 + 1) = (8 7 6 5 4 3 2 1) + 9. This expression must be divisible by 3, since both of its terms are divisible by 3. Therefore, it is not a prime number. Since we found one case that gives a prime and one case that gives a non-prime, statement (2) is NOT sufficient. (1) and (2) INSUFFICIENT: since the number 2 gives a prime, and the number 8 gives a nonprime, both statements taken together are still insufficient. The correct answer is E. 11. (2) Takeaway #1: when you plug numbers on a DS problem, YOUR GOAL IS TO PROVE THAT THE STATEMENT IS INSUFFICIENT. Therefore, as soon as you get a 'yes' answer, you should be TRYING to get a 'no' answer to go

along with it; and, as soon as you get a 'no' answer, you should be TRYING to get a 'yes' answer to go along with it. Statement (2) you need to pick numbers such that x + y > z, per this statement. First, pick a completely random set of numbers that does this: how about x = 1, y = 1, z = 0. These numbers give a YES answer to the prompt question, since 1^4 + 1^4 is indeed greater than 0^4. Now remember: your goal is to prove that the statement is INSUFFICIENT. This means that we have to try for a 'no' answer. This means that we have to make z^4 as big as possible, while still obeying the criterion x + y > z. Fortunately, this is somewhat simple to do: just make z a big negative number. Try x = 1, y = 1, z = -100. In this case, x + y > z (satisfying statement two), but x^4 + y^4 is clearly less than z^4, so, NO to the prompt question. Insufficient. Statement (2) you need to pick numbers such that x^2 + y^2 > z^2, per this statement. First, pick a completely random set of numbers that does this: how about x = 1, y = 1, z = 0 (the same set of numbers we picked last time). These numbers give a YES answer to the prompt question, since 1^4 + 1^4 is indeed greater than 0^4. Now remember: your goal is to prove that the statement is INSUFFICIENT. This means that we have to try for a 'no' answer. This means that we have to make z^4 as big as possible, while still obeying the criterion x^2 + y^2 > z^2. Unfortunately, this isn't as easy to do as it was last time; we can't just make z a huge negative number, because z^2 would then still be a giant positive number (thwarting our efforts at obeying the criterion). So, we have to finesse this one a bit, but the deal is still to make z as big as possible while still obeying the criterion. Let's let x and y randomly be 3 and 3. Then x^2 + y^2 = 18; we need z^2 to be less than this, but still as big as possible. So let's let z = 4 (so that z^2 = 16, which is pretty

close). With these numbers, x^4 + y^4 = 162, which is much less than z^4 = 256. Therefore, NO to the prompt question, so, insufficient. Answer = e. Takeaway #2: if a statement is sufficient, then you WILL be able to PROVE that it is, algebraically or with some other form of theory. In other words, you'll never get a statement that's sufficient, but for which you can only figure that out by number plugging. It's obvious that you can get a YES answer to the question; all you have to do is take ridiculously big numbers for x and y, and a small number for z. for instance, x = y = 100, z = 0, satisfy both statements, and clearly give a YES answer. So, you're trying for a NO answer. Try to make Z as big as possible while still satisfying the criteria (i.e., less than x^2 + y^2). Let's let x = y = 3 then to satisfy both statements, we need z^2 less than 18, and z less than 6. We'll take z = 4, which is pushing the limit of the first one. In this case, then, x^4 + y^4 = 81 + 81 = 162, but z^4 = 256, giving a NO answer. Insufficient Answer = e 12. to get at the heart of this problem, you must realize that "greater" is NOT the same thing as "farther away from zero". for positive numbers, these two concepts are the same, but they are not so for negative numbers (for which the the greater number is actually closer to zero). that realization is the crux of this problem. the SUM is the same for each of the two sets. therefore, since average = (sum) / (# of data points), the average will be CLOSER TO ZERO if there are more data points. the problem is that this doesn't mean that the average is lower. if the sum is negative, then just the opposite will occur. examples: if S = 2, 2, 2 and T = 3, 3, then the sums are both 6, the average of S is less (2 vs. 3), and S has more integers. if S = -3, -3 and T = -2, -2, -2, then the sums are both -6, the average of S is less (-3 vs. -2),

and S has fewer integers. so (1) is insufficient. From (1) above, (2) is also insufficient. together: insufficient as nothing extra is added. answer = (e).

13. (1) gives us a unique answer there is no need to calculate it though whatever the answer, it will be either < 8 or = 8 or > 8 in all the three situations we will get a confirmed YES or confirmed NO. If r 8, we will get NO if r > 8, we will get YES. So (1) is sufficient. (2) 1.08^2 = 116.64 and 1.07^2 = 114.49 For (1+r/100)^2 > 1.15 :1. Assume (1+r/100)^2 = 1.16 solving for 'r', we get r < 8 2. Assume (1+r/100)^2 = 1.44 solving for 'r', we get r = 12 (greater than 8) So clearly insufficient since we get 2 different answers for the scenarios mentioned. Ans. A 14. Notice that we're dealing with a fractional inequality, which, worse yet, CAN'T be multiplied by the common denominator (since we don't know the sign of that denominator). Therefore, pick numbers. Just be careful to pick APPROPRIATE numbers. i.e., this problem contains sums and differences, as well as sign considerations, so i would pick: * POSITIVES AND NEGATIVES (as allowed by the statements) * DIFFERENT RELATIVE SIZES (i.e., "bigger" and "smaller" numbers) this is important because of addition and subtraction. so, for statement (1), i would pick: 1, 2 2, 1 1, 2 2, 1

1, 0 for statement (2), i would pick: 1, 2 2, 1 1, 2 2, 1 0, 1 for "together" i would look at the two common elements, which are (1, 2) and (2, 1). note that this is a lot of plugins, but you don't wind up trying them all you STOP as soon as you get "insufficient". GMAT's answer is (E) this is a difficult problem, because it resists simple algebra. you CANNOT multiply through by the denominator (x + y), because the sign of that denominator is unknown. therefore, you have to leave the problem as written (ugly as it may be). since there's no simple algebraic solution, a fallback is to recognize the types of numbers that are important in the problem, and try numbers across those categories. there are two things that matter in this problem (as may be deduced from an inspection of the problem + experience with these sorts of things): 1. positive vs. negative 2. the relative magnitudes of x and y let's try numbers across both of these categories. statement (1) x must be positive, but y could be positive or negative, and smaller or bigger (or the same) in magnitude. if x = 1 and y = 2 > answer = NO if x = 2 and y = 1 > answer = NO if x = 1 and y = 2 > answer = NO if x = 2 and y = 1 > answer = YES

insufficient [at this point you could notice that the last two examples also satisfy statement 2, and therefore satisfy statements 1 and 2 together. this fact proves that the answer is E right now, and you're done. if you don't notice this (most students won't), then go on.] statement (2) y must be negative, but x could be positive or negative, and smaller or bigger (or the same) in magnitude. if x = 1 and y = 2 > answer = NO if x = 2 and y = 1 > answer = NO if x = 1 and y = 2 > answer = NO if x = 2 and y = 1 > answer = YES insufficient together x must be positive and y must be negative, but the relative magnitudes can go any way (bigger/smaller/same) if x = 1 and y = 2 > answer = NO if x = 2 and y = 1 > answer = YES insufficient ans (e) 15. JUST PLUG IN NUMBERS. statement (1) let's just PICK A WHOLE BUNCH OF NUMBERS WHOSE GCF IS 2 and watch what happens. let's try to make the numbers diverse. say, 4 and 6 6 and 8 8 and 10

10 and 12 ... 4 and 10 6 and 14 6 and 16 8 and 18 8 and 22 ... in all nine of these examples, the remainders are greater than 1. in fact, there is an obvious pattern, which is that they're all even, since the numbers in question must be even. in fact, i just thought of this, which is a much nicer, more ground-level approach to statement one: in statement 1, both m and p are even. therefore, the remainder is even, so it's greater than 1. done. sufficient. -statement (2) just pick various numbers whose lcm is 30. notice the numbers selected above: 5 and 6 --> remainder = 1 10 and 15 --> remainder = 5 > 1 insufficient. ans (a) 16. this is another problem about a topic that is one of the darlings of the test authors: namely, sets of consecutive integers, especially as pertaining to the averages of such sets. here is the fact that you absolutely must know about these sets:

the mean and the median of a set of consecutive integers are the same; both of them are the middle number (for a set with an odd # of numbers in it) or halfway between the two middle numbers (for a set with an even # of numbers in it). -remember that average = sum / number of data points. you should be ultra-aware of this relationship; the vast majority of problems about the sum of a set are really concerned with the average - and vice versa. they are tricky, those test writers. let 'X' stand for the sum of each of the sets. (1) is clearly insufficient, as we know nothing whatsoever about set t. still, take the time to interpret it: it says that the middle number of set s is 0, which also means that the sumof the elements in set s is 0 (by the fact above). (2) using the fact above, we have that the average (whether mean or median - they're the same) of the numbers in set s is X/5, and the average (again, mean or median) of the numbers in set t is X/7. it's tempting to say 'sufficient' here, because at first glance X/5 and X/7 appear to be necessarily different, but they aren't: in the singular case X = 0, the two will be identical. therefore, insufficient. (together) this tells us that X = 0, which means that the median of both sets is 0/7 = 0/5 = 0. sufficient. "the sum of these two sets(sets of consecutive integers) will be equal only when the sum is zero. " is INCORRECT LOGIC. Ex: Set S could be 5,6,7,8 & 9 while Set T could be 2,3,4,5.6.7 & 8 and these sets have equal sums. There are umpteen other examples. 17. If x is +ve, y is +ve

If x is ve, y is 0 If x is 0, y is 0.

So y is not negative For 1, x < 0, x + |x| = 0 For 2, y<1, noticed that y is an integer, y only can be 0. Answer is D 18. (1) INSUFFICIENT: Since this equation contains two variables, we cannot determine the value of y. We can, however, note that the absolute value expression |x2 4| must be greater than or equal to 0. Therefore, 3|x2 4| must be greater than or equal to 0, which in turn means that y 2 must be greater than or equal to 0. If y 2 > 0, then y > 2. (2) INSUFFICIENT: To solve this equation for y, we must consider both the positive and negative values of the absolute value expression: If 3 y > 0, then 3 y = 11 y = -8 If 3 y < 0, then 3 y = -11 y = 14 Since there are two possible values for y, this statement is insufficient. (1) AND (2) SUFFICIENT: Statement (1) tells us that y is greater than or equal to 2, and statement (2) tells us that y = -8 or 14. Of the two possible values, only 14 is greater than or equal to 2. Therefore, the two statements together tell us that y must equal 14. The correct answer is C.

19. (1) INSUFFICIENT: We are told that 5n/18 is an integer. This does not allow us to determine whether n/18 is an integer. We can come up with one example where 5n/18 is an integer and where n/18 is NOT an integer. We can come up with another example where 5n/18 is an integer and where n/18 IS an integer. Let's first look at an example where 5n/18 is equal to the integer 1.

5n If 18 = 1, then

n 18

1 5

. In this case n/18 is NOT an integer.

Let's next look at an example where 5n/18 is equal to the integer 15.

5n If 18 = 15, then

n 18

= . In this case n/18 IS an 3 integer.

Thus, Statement (1) is NOT sufficient. (2) INSUFFICIENT: We can use the same reasoning for Statement (2) that we did for statement (1). If 3n/18 is equal to the integer 1, then n/18 is NOT an integer. If 3n/18 is equal to the integer 9, then n/18 IS an integer. This tells us n is a multiple of 6. (1) AND (2) SUFFICIENT: If 5n/18 and 3n/18 are both integers, the difference of 5n/18 and 3n/18 will also be integer (integer integer = integer) So 5n/18 3n/18 = 2n/18 = n/9 = integer n is a multiple of 9 So n is a multiple of both 6 and 9 so n is a multiple of 18. Another way to understand this solution is to note that according to (1), n = (18/5)*integer, and according to (2), n = 6*integer. In other words, n is a multiple of both 18/5 and 6. The least common multiple of these two numbers is 18. In order to see this, write 6 = 30/5. The LCM of the numerators 18 and 30 is 90. Therefore, the LCM of the fractions is 90/5 = 18. Again, the correct answer is C.

20. This question cannot be rephrased since it is already in a simple form. (1) INSUFFICIENT: Since x2 is positive whether x is negative or positive, we can only determine that x is not equal to zero; x could be either positive or negative. (2) INSUFFICIENT: By telling us that the expression x |y| is not a positive number, we know that it must either be negative or zero. If the expression is negative, x must be negative (|y| is never negative). However if the expression is zero, x or y could be zero. (1) AND (2) INSUFFICIENT: We know from statement 1 that x cannot be zero, however, there are still two possibilites for x: x could be positive (y is zero), or x could be negative (y is anything). The correct answer is E.

21. The first 14 terms can be 130 each and the 15th term can be a large number so that the average is 150. So II and III are not necessarily correct. For I, In order to solve the question easier, we simplify the numbers such as 150, 000 to 15, 130,000 to 13, and so on. I. Median is 13, so, the greatest possible value of sum of eight prices that no more than median is 13*8=104. Therefore, the least value of sum of other seven homes that greater than median is (15*15-104) /7 = 17.3 > 16.5. It's true. Answer: only I must be true. 22. Initially water = 99, solid = 1 Now solid = 1, which should be 2% and the remaining water should be 98% Suppose x amount of water has evaporated

So we have Solid/Water = 2/98 or 1/(99-x) = 2/98 or x = 50. Ans. E

23. If x = 0.1, then x^2 < 2x < 1/x (so 1 is possible) If x = 0.9, then x^2 < 1/x < 2x (so 2 is possible) (1) x^2<2x<1/x This means that x^2<2x so divide by x to get x<2. The second one tells you that 2x<1/x which simplifies to x < 1/sqrt(2). These can obviously both be satisfied at the same time, so (1) works. (2) x^2<1/x<2x This means that x^2<1/x which gives x^3<1, or x<1. The second half gives you 1/x<2x or 1<2(x^2) or x>1/sqrt(2). So any number that satisfies 1/sqrt(2)<x<1 will work. (3) 2x<x^2<1/x. The first part gives 2x<x^2 or x>2. The second half gives x^2<1/x or x^3<1 or

x<1. Since the regions x>2 and x<1 do not overlap, (3) can not be satisfied. The Answer choice is (4), 1 and 2 only

24.

THIS IS AN 800 LEVEL PROBLEM.

Innovative approach: Imagine right angled triangle with sides 12. As the smallest right triangle is 3, 4, 5 and 12 is a multiple of 3 and 4 both, we may make

3, 4, 5 MULTIPLIED by 3 = 9, 12, 15 and 3, 4, 5 MULTIPLIED by 4 = 12, 16, 20.

Now lets check the perimeter. 16 + 9 + 20 + 15 = 60, so these are the correct combinations. Area ratio = (L1/L2)2 = (4/3)2 = 16/9.

Short-cut: The area of two similar figures will be in the ratio of squares of sides. So the best guess answer is 16/9.

25.

Innovative Approach

We can use the Pythagorean Theorem to see the ED = 9. Imagine that the left triangle were to be shifted and made the way it is mentioned in the diagram below: then we just have to find the area of the rectangle with sides 12 and 9 ans. 12*9 = 108.

Detailed Solution:

By sketching a drawing of trapezoid ABDC with the height and diagonal drawn in, we can use the Pythagorean theorem to see the ED = 9. We also know that ABDC is an isosceles trapezoid, meaning that AC = BD; from this we can deduce that CE = FD, a value we will call x. The area of a trapezoid is equal to the average of the two bases multiplied by the height. The bottom base, CD, is the same as CE + ED, or x + 9. The top base, AB, is the same as ED FD, or 9 x. Thus the average of the two bases is . Multiplying this average by the height yields the area of the trapezoid: The correct answer is D.

Anda mungkin juga menyukai